You are on page 1of 53

QUIZ 1 - GFORMS

1. A programmable electronic device that accepts raw data as input and processes it with a set of
instructions (a program) to produce the result as output.
A. Software B. Hardware C. Computer D. Peripherals

2. Believed to be the first computer


A. Modem B. Abacus C. Mobile Phones D. USB

3. Collection of unprocessed items which can include text, numbers, images, audio and video.
A. Information B. Table C. Data D. Device

4. Involves reducing the electricity consumed and environment waste generated when using a computer.
A. Sustainability software B. Environmental software C. Planet computing D. Green computing

5. A collection of computers and devices connected together, often wirelessly via communication devices
and transmission media.
A. Web B. Property, Plant and Equipment C. Communication D. Network

6. Business-wide system that interconnects all the processes in the business.


A. Software system B. Enterprise system
C. Hardware system D. Electronic communication

7. A server controls access to the hardware, software, and other resources on a network and provides a
centralized storage area for programs, data and information. Where would an accountant normally
classify a server?
A. Intangible Assets B. Office Furnitures and Fixtures
C. Land D. Property Plant and Equipment

8. An accounting information system that processes the financial transactions and events to produce
accounting and financial reports using computer and other digital tools and software.
A. Electronic Trial Balance B. Computerised Accounting System
C. International Accounting Software D. Data Information System

9. A company provides communication allowance to employees in managerial role. Communication


allowance includes payment of monthly post-paid plans to telco companies. The employee chooses the
mobile device which they will receive for free as part of the enrolment in the post-paid plan offered by
telco companies. How should the company record the payment to telco companies?
A. The mobile device provided to employees should form part of the PPE of the company
depending on the capitalization policy.
B. The payment to telco should be treated as communication expense of the company.
C. The company should not record PPE but instead recognize pre-paid assets for the amount
being paid to telco companies.
D. The company should not record PPE but instead recognize an expense as part of
employee benefits.

10. Company A is aware that their finance department is employing too many accountants as against to
the activities being performed. The management decided to perform current state assessment and found
out that there are processes they can automate by deploying a new system. Who are the best persons to
do the end user testing of the new system to be deployed.
A. The IT project team since they know the programming language used.
B. The Finance department who will heavily use the new system since they know the principles in
performing the finance processes.
C The management of the company since they will decide whether they will use the new
system or not.
D. Someone independent from the company so that we can rely on the integrity of the results of
the testing.

11. Finance department of Company A is having difficulty reconciling the Accounts Receivable subledgers
versus general ledgers. The total of subledgers net realizable value is less than the net realizable value
reported in the general ledger. Company A no longer look into the differences but instead, the
management decided to automate the reconciliation process so that it will be faster for them close the
books on a monthly basis.
A. Automation of the reconciliation between subledger and general ledger will reduce the time it
will take for a person to perform the process.
B. Company A will no longer encounter differences in SL vs GL.
C. The differences between SL and GL will still remain since Company A did not really look
into the causes of the differences.
D. It will be more beneficial for Company A to automate the process and then resolve the issue
rather than resolving the issue and then automating the process.

12. Company A bought a license to use Microsoft Outlook as the company's information manager
software system. Company A is required to pay an annual fee for the use of the software. How should
Company A record the payment for the use of the software?
A. Company A should recognize intangible asset since they bought the license for Microsoft
Outlook
B. Company A should recognize prepaid asset if the payment was made at the beginning
of the year.
C. Company A should recognize PPE for the purchase of the license from Microsoft Outlook.
D. Company A should directly recognize an expense for the whole annual payment of the
purchase of the license.

13. Company A bought several units of laptop for the use of their employees. It is the policy of the
company to allow employees to work from home or in any part of the world using the company provided
laptop. How should the purchase of the laptop be treated in the books of Company A?

A. Should be recognized as employee benefits since the employees are allowed to work
anywhere.
B. Should be recognized as office expense upon purchase of the laptops.
C. The company should recognize Advances to Employees since the laptops are being used by
employees.
D. The company should recognize PPE for the purchase and depreciate in accordance
company's policy PPE policy.

14. Holds data, instructions, and information for future use


A. Internet B. Web C. Hard disc D. Storage

15. Consists of a special-purpose chip and electronic circuits that control the transfer of data, instructions,
and information from a disk to and from the system bus and other components in the computer.
A. Hard board B. Hard disk controllers C. Mouse D. Computer

16. Enable mobile users to transport digital photos, music, or files to and from mobile devices and
computers or other devices.
A. Networks B. Cables C. Memory Cards D. USB

17. An internet service that provides hard disk storage to computer users.
A. World Wide Web B. Cloud storage C. Internet D. BUS

18. A type of optical storage media that consists of a flat, round, portable disc made of metal, plastic, and
lacquer.
A. Hard drive B. Flash drive C. Optical disc D. Optical drive

19. A set of programs containing instructions that work together to coordinate all the activities among
computer hardware resources.
A. Operating System B. Computer Instructions
C. Automated Machines D. Accounting Software

20. A type of system software that allows a user to perform maintenance type tasks, usually related to
managing a computer, its devices, or its programs.
A. Stand-alone B. Utility program C. Server D. Tools

21. Describes a process in which two or more computers or devices transfer data, instructions, and
information
A. Networks B. Connections C. Data transfer D. Computer communications

22. A wireless network that provides Internet connections to mobile computers and devices Wi
A. Wireless connection B. Hot spot C. Cold spot D. Router

23. A navigation system that consists of one or more earth based receivers that accept and analyze
signals sent by satellites in order to determine the receiver’s geographic location.
A. Software B. Satellite C. GPS D. USB

24. A collection of computers and devices connected together via communications devices and
transmission media.
A. Cable B. Computer Network C. WiFi D. Connection

25. An internal network that uses Internet technologies.


A. Internal internet B. Group network C. Intranet D. Data network

26. An adapter card, PCCard, ExpressCard module, USB network adapter, or flash card that enables a
computer or device that does not have networking capability to access a network
A. Network Card B. Modem C. Router D. Satellite

27. A handheld hardware input device that controls a cursor in a GUI for pointing, moving, and selecting
text, icons, files, and folders on your computer
A. Flash drive B. Portable printer C. Mouse D. USB data
28. Which of the following statements accurately describes the impact that automation has on the controls
normally present in a manual system?
A. Transaction trails are more extensive in a computer-based system than in a manual system
because there is always a one-for-one correspondence between data entry and output.
B. Responsibility for custody of information assets is more concentrated in user departments in a
computer-based system than it is in a manual system.
C. Controls must be more explicit in a computer-based system because many processing
points that present opportunities for human judgment in a manual system are eliminated.
D. The quality of documentation becomes less critical in a computer-based system than it is in a
manual system because data records are stored in machine-readable files.

29. The two broad groupings of information systems control activities are general controls and application
controls. General controls include controls
A. Relating to the correction and resubmission of faulty data.
B. For developing, modifying, and maintaining computer programs.
C. Designed to assure that only authorized users receive output from processing.
D. Designed to ensure that all data submitted for processing have been properly authorized.

30. The practice of maintaining a test program library separate from the production program library is an
example of
A. An organizational control. B. Physical security.
C. An input control. D. A concurrency control.

31. In traditional information systems, computer operators are generally responsible for backing up
software and data files on a regular basis. In distributed or cooperative systems, ensuring that adequate
backups are taken is the responsibility of
A. User management. B. Systems programmers. C. Data entry clerks. D. Tape librarians.

32. An organization’s computer help-desk function is usually a responsibility of the


A. Applications development unit. B. Systems programming unit.
C. Computer operations unit. D. User departments.

33. Most large-scale computer systems maintain at least three program libraries: production library (for
running programs); source code library (maintains original source coding); and test library (for programs
which are being changed). Which of the following statements is correct regarding the implementation of
sound controls over computer program libraries?
A. Only programmers should have access to the production library.
B. Users should have access to the test library to determine whether all changes are properly
made.
C. Only the program librarian should be allowed to make changes to the production library.
D. The computer operator should have access to both the production library and the source code
library to assist in diagnosing computer crashes.

34. Which of the following would not be appropriate to consider in the physical design of a data center?
A. Evaluation of potential risks from railroad lines and highways.
B. Use of biometric access systems.
C. Design of authorization tables for operating system access.
D. Inclusion of an uninterruptible power supply system and surge protection.
35. What type of information system uses communications capabilities to make needed data and
computing capability available to end users at separate locations?
A. Distributed processing system. B. Time-sharing system.
C. Online processing system. D. Personal computing system.

36. Even though an organization is committed to using its mainframe for its manufacturing plant
operations, it has been looking for ways to downsize other applications. The purpose of downsizing is to
A. Improve reliability. B. Improve security. C. Reduce complexity. D. Decrease costs.

37. A firm has recently converted its purchasing cycle from a manual process to an online computer
system. Which of the following is a probable result associated with conversion to the new automatic
system?
A. Processing errors are increased. B. The firm’s risk exposures are reduced.
C. Processing time is increased. D. Traditional duties are less segregated.

38. A small client recently put its cash disbursements system on a server. About which of the following
internal control features would an auditor most likely be concerned?
A. Programming of the applications are in BASIC, although COBOL is the dominant, standard
language for business processing.
B. The server is operated by employees who have cash custody responsibilities.
C. Only one employee has the password to gain access to the cash disbursement system.
D. There are restrictions on the amount of data that can be stored and on the length of time that
data can be stored.

39. Which of the following controls most likely could prevent computer personnel from modifying
programs to bypass programmed controls?
A. Periodic management review of computer utilization reports and systems documentation.
B. Segregation of duties within computer for computer programming and computer
operations.
C. Participation of user department personnel in designing and approving new systems.
D. Physical security of computer facilities in limiting access to computer equipment.

40. A corporation has changed from a system of recording time worked on clock cards to a computerized
payroll system in which employees record time in and out with magnetic cards. The computer system
automatically updates all payroll records. Because of this change
A. A generalized computer audit program must be used.
B. Part of the audit trail is altered.
C. The potential for payroll-related fraud is diminished.
D. Transactions must be processed in batches.

41. In the organization of the information systems function, the most important separation of duties is
A. Not allowing the data librarian to assist in data processing operations.
B. Assuring that those responsible for programming the system do not have access to
data processing operations.
C. Having a separate information officer at the top level of the organization outside of the
accounting function.
D. Using different programming personnel to maintain utility programs from those who maintain
the application programs.
42. Information processing made possible by a network of computers dispersed throughout an
organization is called
A. Online processing. B. Interactive processing. C. Time sharing. D. Distributed data processing.

43. The concept of timeliness of data availability is most relevant to


A. Payroll systems. B. Manual systems. C. Computerized systems. D. Online systems.

44. What type of computer system is characterized by data that are assembled from more than one
location and records that are updated immediately?
A. Personal computer systems. B. Midrange computer systems.
C. Batch processing systems. D. Online, real-time systems

45. If a workstation contains a processor, monitor, printer, storage, and communications capabilities, it is
said to be a(n)
A. Dumb workstation. B. Intelligent workstation.
C. Noninteractive workstation. D. Desktop publishing workstation.

46. Application control objectives do not normally include assurance that


A. Authorized transactions are completely processed once and only once.
B. Transaction data are complete and accurate.
C. Review and approval procedures for new systems are set by policy and adhered to.
D. Processing results are received by the intended user.

47. In a large organization, the biggest risk in not having an adequately staffed information center help
desk is
A. Increased difficulty in performing application audits.
B. Inadequate documentation for application systems.
C. Increased likelihood of use of unauthorized program code.
D. Persistent errors in user interaction with systems.

48. Regardless of the language in which an application program is written, its execution by a computer
requires that primary memory contain
A. A utility program. B. An operating system. C. Compiler. D. Assembly.

49. Misstatements in a batch computer system caused by incorrect programs or data may not be detected
immediately because
A. Errors in some transactions may cause rejection of other transactions in the batch.
B. The identification of errors in input data typically is not part of the program.
C. There are time delays in processing transactions in a batch system.
D. The processing of transactions in a batch system is not uniform.

50. Which of the following statements most likely represents a disadvantage for an entity that keeps data
files on a server rather than on a manual system?
A. Attention is focused on the accuracy of the programming process rather than errors in
individual transactions.
B. It is usually easier for unauthorized persons to access and alter the files.
C. Random error associated with processing similar transactions in different ways is usually
greater.
D. It is usually more difficult to compare recorded accountability with the physical count of assets.
QUIZLET - MGA KASAMA NUNG MGA QUESTIONS SA QUIZ

1. Which of the following conveys meaning and is useful to users?


a. Data c. Input
b. Information d. Output

2. You want to interact with a touch screen. Which of the following do you use?
a. telemetry c. apps
b. gestures d. servlets

3. Which of the following is an example of data on a grade report?


a. total semester hours c. course names
b. grade point average d. total credits

4. If a manufacturer is looking for memory that is very durable and shock resistant, which kind of
technology might it use in its laptops?
a. SSDs c. platters
b. hard disks d. DVD drives

5. If you are looking for an online social network that allows you to stay current with the daily
activities of those you are following, which of the following is the best match for you?
a. Facebook c. LinkedIn
b. Twitter d. Digg

6. Which of the following allows access to high-speed Internet services through the cable television
network?
a. cable transceiver c. cable receiver
b. cable modem d. cable dialer

7. Which of the following is equal to approximately one million characters?


a. kilobyte c. gigabyte
b. megabyte d. terabyte

8. What is the purpose of an Internet Protocol address (IP address)?


a. It specifies whether a computer is using a broadband network or a direct-dial network.
b. It differentiates between a server computer and a client computer and contains codes to identify
whether the computer can send data or receive data.
c. It contains the protocol schema used to identify whether the computer is on a wired or wireless
network.
d. It uniquely identifies the location of each computer or device connected to the Internet.
.
9. In Internet terminology, what is the term, google.com, called?
a. domain name c. webpage protocol
b. Internet service provider d. TLD

10. Which of the following best describes e-commerce?


a. a customer (consumer) experience requiring the Internet, cash, a shopping cart, and videos
b. a business transaction that occurs over an electronic network such as the Internet
c. the sale of electronic goods and services that enable an Internet connection
d. a transaction between two businesses that are bartering and auctioning goods and services via secure
telephonic connections

11. Which of the following identify the five steps in web publishing?
a. load plug-ins; deploy the website; maintain the website; update the website; upgrade the web server
b. plan the website; design the website; create the website; host the website; maintain the website
c. plan, analyze, and design the website; plan and purchase servers and server software; create and
deploy the website; test the website for both commercial and consumer use; modify the website based on
analytics developed through the website use
d. plan the website; design the website; create and deploy the website; update the website; create usage
analytics for further website enhancements

12. Which of the following is the term for unsolicited mail messages?
a. e-junk c. malware
b. spam d. mail output

13. Which of the following are the fastest, most powerful computers — and the most expensive?
a. desktop computers c. midrange servers
b. notebook computers d. supercomputers

14. You want to find a device that can connect to your mobile device and enable you to print. Which
of the following do you use?
a. embedded computer c. telemetry
b. peripheral d. integrated system

15. Which of the following do you need to verify that you are the holder of the bank card at your local
bank branch?
a. skimmer c. PIN
b. account code d. transaction ID

16. You are storing a new worksheet online for members of your project team to be able to share
access to it. What kind of computing is this an example of?
a. cloud c. disperse
b. digital d. liberated

17. A friend of yours was talking on a mobile phone and her attention was diverted from something
that was happening around her while she was on the phone. What phenomenon does this
illustrate?
a. inattentional blindness c. distracted vision
b. failure to integrate d. visual inactivity

18. Which of the following is a request for specific data from a database?
a. query c. resample
b. formula d. Function

19. Which of the following is a utility that detects and protects a computer from unauthorized
intrusions?
a. personal firewall c. threat neutralizer
b. antihacker d. protection utility
20. Which of the following are you, as a marketing manager, most likely to use to schedule the
processes required in a new advertising campaign you are running?
a. personal finance c. calendar management
b. project management d. software suite

21. An engineer who needs to manage new product development might use what type of software?
a. project management software
b. engineering management software
c. product development management software
d. process management software

22. Why is a fragmented disk slower than one that is defragmented?


a. A fragmented disk contains temporary Internet files and unused program files that need to be deleted.
b. A fragmented disk contains programs that were deleted using Windows Explorer.
c. A fragmented disk has many files stored in noncontiguous sectors.
d. A fragmented disk has no index.

23. If you follow the advice of your instructor and use CBT to learn about programming languages,
how would you learn?
a. You would access content by teachers from a web site designed to reinforce what you already know
about programming.
b. You would answer a series of questions from your instructor and then, based on your answers, your
instructor would assign books to read and programs to review.
c. Typically you would employ self-paced instruction to use and complete exercises presented on your
computer.
d. You would use simulations.

24. Which of the following has the same intent as a cracker, but does not have the technical skills and
knowledge?
a. hacker c. cyberextortionist
b. script kiddie d. cyberterrorist

25. Which of the following is a technique intruders use to make their network or Internet transmission
appear legitimate to a victim computer or network?
a. rebuking c. transmitting
b. spoofing d. victimizing

26. Which of the following can be used to create a software registration number, sometimes illegally?
a. BSA c. keygen
b. decryption key d. public key

27. What is an EULA intended to do?


a. license software for individual use
b. outline computer activities for which the computer and network may or may not be used
c. contract with an outside organization to provide security and recovery services for a network
d. establish a network of computers, such as a homegroup

28. Which of the following statements is not true about Internet and network attacks?
a. No central administrator is present on the Internet.
b. It is usually the responsibility of the network administrators to take measures to protect a network from
security risks.
c. Information kept on an organization's premises has a higher security risk than information transmitted
over networks.
d. Internet and network attacks that jeopardize security include malware, botnets, denial of service
attacks, back doors, and spoofing.

29. You have encountered a technology that is effective in blocking computer-generated attempts to
access a website because it is generally easy for humans to recognize distorted characters. What
is this technology?
a. CAPTCHA c. two-step verification
b. CERT/CC d. serial authentication.

Which of the following interprets and carries out the basic instructions that operate a computer?
a. control unit c. CPU
b. binary device d. compressor

30. The faster the clock speed, the more of these the processor can execute per second. What are
they?
a. registers c. arguments
b. functions d. instructions

31. Your business is looking to rely on cloud computing services to provide system backup and the
archiving of earlier versions of files, for example. What is the term for this?
a. Storage as a Service c. Data as a Service
b. Software as a Service d. Platform as a Service

32. When your friend turns off her computer, what kind of memory loses its contents?
a. partial
b. static L1 ROM
c. digital fixed memory when installed on memory modules
d. volatile

33. Computer memory and computer storage are available in varying sizes. Which of the following is
the largest amount of memory or storage?
a. kilobyte (KB) c. megabyte (MB)
b. terabyte (TB) d. gigabyte (GB)

34. Which of the following describe computer memory?


a. electronic components that store, among other things, instructions waiting to be executed by the
processor
b. chips stored on the computer processor board that interact with both the system unit and the processor
in order to extract the data required for processing
c. a series of digital devices embedded within computers that interprets commands from computer users
and remembers them for future reference in order to increase the processing power of a computer
d. manufactured devices that store a single digital address at a time in order to facilitate an orderly
transfer of data within a computer
35. What is the definition of software?
a. a collection of unprocessed text, numbers, images, audio, and video
b. a series of related instructions that tells a computer or mobile device the tasks to perform and how to
perform them
c. an instruction issued by replying to a question that is displayed
d. an instruction that causes a single specific action to be performed

36. Which of the following is a technology that reads hand-drawn marks such as small circles or
rectangles?
a. OMR c. OCR
b. TCR d. MICR

37. When your sister goes to work, what scientific discipline is used in the design of her workplace to
incorporate comfort, efficiency, and safety for her?
a. human entomology c. form technics
b. factoronomics d. ergonomics

38. Your instructor has asked you to perform some research regarding a computer's capability of
distinguishing spoken words. What is the best search term to use for your research?
a. voice recognition c. speech differentiation
b. digital voice input d. speech identification

39. Which of the following items are required in order for your company to conduct a
videoconference?
a. network, television screen, tablet PCs, microphone
b. videoconferencing software or web app, speakers, communications-enabled eyeglasses for each
participant
c. video camera, digital pens, microphone
d. videoconferencing software or web app, microphone, video camera

40. When a user types data on a keyboard, what is occurring?


a. The user is using an input device to enter input into the memory of the computer.
b. The user is issuing a command that is stored in the memory of the computer and then is transformed
into input data for use with a program.
c. The user is employing an input device (the keyboard) to tell the computer what tasks to perform and
how to perform them.
d. The user is entering information that will be transferred to the CPU for interpretation and then to
storage so that the information will be available when needed.

41. Which of the following terms is the number of bytes (characters) a storage medium can hold?
a. resolution c. capacity
b. dimensionality d. retention

42. Some computers improve hard disk access time by using which of the following, which consists of
memory chips that store frequently accessed items?
a. warehouse c. base
b. disk cache d. home

43. You are looking for the best protection against data loss. Which of the following will you use?
a. archival backup c. selective backup
b. differential backup d. partial backup

44. You are looking to implement a high-speed network with the sole purpose of providing storage to
other attached servers. Which of the following will you implement?
a. SCSI c. EIDE
b. SAN d. eSATA

45. Which of the following statements is not true about solid state media?
a. Flash memory chips are a type of solid state media.
b. They consist entirely of electronic components, such as integrated circuits.
c. They contain moving parts.
d. They are more durable and shock resistant than other types of media.

46. Items on a storage medium remain intact even when power is removed from the computer. What
term describes this characteristic?
a. nonversatile c. nonviolate
b. nonvolatile d. nonviolent

47. Which of the following kinds of software consists of the programs that control or maintain the
operations of a computer and its devices?
a. system c. management
b. application d. program

48. Which of the following allows a computer or mobile device's processor to continue interpreting
and executing instructions while a printer prints?
a. paging c. spooling
b. thrashing d. buffing

49. When a computer is running multiple programs concurrently, what is the program that is in the
foreground called?
a. main program c. active program
b. open program d. primary program

50. What happens when the computer is thrashing?


a. A hacker is attempting to circumvent antivirus software and firewalls.
b. Items are being released from being stored in memory because the processor no longer requires them.
c. The computer is exchanging data between memory and storage.
d. The operating system spends too much of its time paging, instead of executing application software.

51. What is the purpose of memory management?


a. to store data in the most efficient manner
b. to optimize the use of RAM
c. to start the computer and operating system quickly
d. to allocate system resources stored in ROM

52. You want to clear the memory on your mobile device. Which of the following is sufficient?
a. cold boot c. new boot
b. warm boot d. soft boot
53. Digital communications describes a process in which two or more computers or devices transfer
all of the following EXCEPT ____.
a. data c. instructions
b. information d. networks

54. The amount of data, instructions, and information that can travel over a communications channel
sometimes is called which of the following?
a. broadband c. baseband
b. latency d. bandwidth

55. Which of the following is the least likely technology PANs would use?
a. Bluetooth c. Wi-Fi
b. peer-to-peer d. special USB cables

56. Which network type connects each computer and device to a central device?
a. a bus network c. a star network
b. a ring network d. a core network

57. Which of the following technologies is your new wireless headset most likely to use?
a. TCP/IP c. IrDA
b. Bluetooth d. RFID

58. Which of the following technologies is the new tollbooth on the local highway most likely to use?
a. TCP/IP c. IrDA
b. Bluetooth d. RFID

59. Which of the following is a collection of data organized in a manner that allows access, retrieval,
and use of that data?
a. database c. spreadsheet
b. server d. webbase

60. A DBMS uses which of the following to perform validation checks?


a. data server c. data warehouse
b. data mart d. data dictionary

61. If you are running a check to ensure that a required field contains data, or the correct minimum
number of characters, what kind of check are you running?
a. completeness c. range
b. alphabetic d. check digit

62. Your database was damaged due to hardware failure. What can you use to restore it?
a. a backup c. restoration hardware
b. imaging software d. a wizard

63. Which term does a user of a relational database use to refer to a record?
a. row c. column
b. table d. attribute
64. Which of the following is true about the database approach?
a. Data in a database is less vulnerable than data in file processing systems.
b. Unauthorized users are unable under any circumstances to gain access to a single database file that
contains personal and confidential data.
c. Business and home users often work with databases because of their numerous advantages.
d. A database is never more complex than a file processing system.

65. Which of the following describes an information system?


a. a network of computers that enables the transfer of information from one computer to another, such as
the Internet
b. a collection of hardware, software, data, people, and procedures that work together to produce quality
information
c. a group of data professionals who work together to enhance communications systems
d. a series of activities such as analysis and design in the process of distributing information on a large
scale

66. You purchase a spreadsheet program for your computer from a vendor who mass-produces
software. What is the term for this type of software?
a. popular c. retail
b. platform d. prepared

67. What is another term for a computer programmer?


a. scripter c. developer
b. coder d. reporter

68. What does an interpreter do?


a. translate program code from a first- to a second-generation language
b. reads a code statement, converts it to one or more machine language instructions, and executes the
instructions
c. includes common programming instructions so a programmer can reuse them, saving time and
reducing error
d. a program that converts the entire source program into machine language and saves it on storage
media for execution later

69. In sequence, traditional system development contains five phases. What are they?
a. planning, analysis, design, implementation, support and security
b. analysis, implementation, support and security, design, planning
c. support and security, design, planning, analysis, implementation
d. implementation, analysis, support and security, planning, design

33. IT systems have dramatically affected many aspects of business. Which of the following is not one of
the changes?
A. Data input into accounting information systems.
B. The way that data is processed in the system.
C. The accounting information that is reported by the system.
D. The outputs of the system.

34. Which of the following statements is false?


A. Technology has allowed many industries to provide better, faster, and higher quality information.
B. Business process must adapt to the new technologies.
C. Business processes, IT systems, and the accounting information system are inextricably linked.
D. IT systems have not had a major impact on the input of data into the accounting information system.

35. The system that captures, records, processes, and reports accounting information is referred to as
a(n):
A. Accounting information system
B. Management information system.
C. System of business processes.
D. Client-server system.

36. Information captured by a system is generated by financial transactions:


A. Within the organization only.
B. Between an organization and its customers only.
C. Between an organization and its vendors only.
D. Within the organization and between an organization and its customers and vendors.

37. A prescribed sequence of work steps completed in order to produce a desired result for an
organization is the definition of:
A. Accounting information system
B. Business process
C. Business transaction
D. Financial statement

38. Which of the following statements, related to a business process, is not a true statement?
A. It has a well-defined beginning and end.
B. Usually takes a long-period of time to complete.
C. Occur so that the organization may serve its customers.
D. Is initiated by a particular kind of event.

39. When a transaction occurs there are systematic and defined steps that take place within the
organization to complete all of the underlying tasks. These "defined steps" are referred to as:
A. Business Processes
B. Financial Transactions
C. Accounting Information Systems
D. Customer Service Arenas

40. The substance of an accounting information system includes:


A. Initiate an event
B. Steps taken to create a business process
C. Capturing, recording, processing and reporting accounting information
D. Selection of client-server computing

41. Which of the following is not one of the general categories of business processes?
A. Revenue Processes
B. Inventory Processes
C. Expenditure Processes
D. Conversion Processes
42. Which of the following correctly states the order of steps in a manual accounting system?
A. Source Documents, Journals, Ledgers, Reports
B. Journals, Source Documents, Ledgers, Reports
C. Source Documents, Ledgers, Journals, Reports
D. Ledgers, Reports, Journals, Source Documents

43. Which of the following statements is true?


A. Most accounting systems in use today are computerized systems.
B. All accounting systems, whether computerized or not, must capture data, process the data, and
provide outputs
C. It is not necessary for an accounting system to maintain summary information if it maintains detail
information.
D. The general ledger will be summarized and then posted to the subsidiary ledger.

44. The choice of accounting information system will depend on all of the following except:
A. The size of the organization
B. The philosophy of management
C. The ability of the company to capture information
D. The nature of its processes

45. The record that captures the key data of a transaction is called:
A. Turnaround document
B. Ledger
C. Journal
D. Source document

46. In order to organize the study of accounting information systems, the authors divided the systems in
place into three categories. Which of the following is not one of those categories?
A. Manual Systems
B. Client-Server Systems
C. Legacy Systems
D. Integrated IT Systems

47. Which of the following statements is true as it relates to manual systems?


A. Few small organizations use computerized accounting systems.
B. Computerized systems rely on some manual record keeping.
C. Most medium sized organizations rely on manual accounting systems.
D. When using a computerized system, it is not necessary to understand the manual system.

48. Which of the following is an example of a source document?


A. Purchase order
B. Employee time card
C. Cash receipts
D. All of the above are examples of source documents.

49. A source document serves important functions in the accounting system. Which of the following is not
one of those functions?
A. Provides the output data for financial reports
B. Serves as part of the permanent audit trail
C. Triggers beginning of business processes
D. Provides the input data to record the transaction

50. An output of the accounting system that can be used as an input in a different part of the accounting
system is referred to as:
A. Round table document
B. Source document
C. Turnaround document
D. Financial report

51. Which of the following provides details for the entire set of accounts used in the organization's
accounting systems?
A. General Ledger
B. Special Journal
C. Subsidiary Ledger
D. General Journal

52. The book of original entry for any transaction not recorded in a special journal is the:
A. Special Ledger
B. General Ledger
C. General Journal
D. Subsidiary Journal

53. Special journals are created, or established, to record specific types of transactions. Which of the
following is not one of the special journals?
A. Fixed Asset Journal
B. Cash Receipts Journal
C. Purchases Journal
D. Payroll Journal

54. The purpose of this item is to maintain the detailed information regarding routine transactions, with an
account established for each entity.
A. Purchases Journal
B. Subsidiary Ledger
C. General Journal
D. General Ledger

55. Which of the following items is not one of the manual records in a manual accounting system?
A. Journals
B. Ledgers
C. Source Documents
D. Trial Balance

56. Many of the accounting software systems consist of modules. Which of the following statements is
false related to these modules?
A. Are separate programs for different business processes
B. Include accounts receivable, payroll, and accounts payable
C. Have essentially the same purpose as special journals and special ledgers
D. Are referred to as special purpose systems
57. An existing system within the organization that uses older technology is called a(n):
A. Manual system
B. Legacy system
C. Client-server system
D. Modern integrated system

58. Which of the following statements does not refer to a legacy system?
A. Includes source documents, journals, and ledgers.
B. Have been in place for may years.
C. Much time has been spent developing, maintaining, and customizing the system.
D. Often based on old or inadequate technology.

59. Organizations are often reluctant to abandon their legacy systems because:
A. The system was customized to meet specific needs.
B. The process to replace the systems is inexpensive.
C. The time involved for replacement is minimal.
D. Information contained on the system is outdated.

60. Which of the following is not one of the advantages of maintaining the legacy systems?
A. The system often supports unique business processes not inherent in generic accounting software.
B. The system contains invaluable historical data that may be difficult to integrate into a new system.
C. The system is well supported and understood by existing personnel who are already trained to use the
system.
D. The system has been customized to meet specific needs of the organization.

61. Which of the following is one of the disadvantages of maintaining the legacy systems?
A. Often support unique business processes not inherent in generic accounting software.
B. May not easily run on new hardware.
C. Contain invaluable historical data.
D. Existing personnel are already trained to use the system.

62. Which of the following is not one of the disadvantages of maintaining the legacy systems?
A. They become difficult to integrate when companies merge or acquire other companies.
B. They are often difficult to modify to make them web based.
C. They are well supported and understood by existing personnel.
D. They lack adequate, up-to-date supporting documentation.

63. Frontware, which adds modern, user friendly screen interfaces to legacy systems are referred to as:
A. Turnaround software
B. Graphical user face
C. COBOL
D. Screen scrapers

64. Instead of completely replacing their systems, organizations often try to use new technology to
enhance existing systems. Which of the following is not one of the approaches taken by the se
organizations?
A. Enterprise application integration
B. Screen scrapers
C. Enterprise resource planning
D. Using interface bridges

65. A set of processes, software and hardware tools, methodologies, and technologies to integrate
software systems is referred to as:
A. Enterprise application integration
B. Client-server interface
C. Screen scrapers
D. Complete integration

66. An use of enterprise application integration would include:


A. Bridge the legacy systems to the new hardware and software.
B. Intended to consolidate, connect, and organize all of the computer applications, data, and business
processes.
C. Allow real-time exchange and management of all the critical information.
D. All of the listed items would be included.

67. Which of the following statements is false?


A. When a modern, integrated system is purchased, they are often modified by the IT staff to meet the
needs of the organization.
B. New programs sold by software development companies are not as user friendly as the legacy
systems.
C. Purchased software has a lower cost, shorter implementation time, and fewer bugs.
D. The modern, integrated systems typically use the latest technology.

68. There were four market segments identified by the authors in chapter 2. Which of the following is not
one of those segments?
A. Small Company Systems
B. Midmarket Company Systems
C. Legacy Company Systems
D. Beginning ERP Systems

69. A company with revenue of $50 million would be most likely to purchase software from which one of
the four market segments?
A. Midmarket Segment
B. Beginning ERP Segment
C. Tier 1: ERP Segment
D. Integrated Segment

70. Quickbooks and Peachtree would be part of which market segment of accounting software?
A. Small Segment
B. Midmarket Segment
C. Beginning ERP Segment
D. Legacy Segment

71. SAP and PeopleSoft are part of which market segment of accounting software?
A. Small Segment
B. Midmarket Segment
C. Beginning ERP Segment
D. Tier 1 ERP Segment

72. Five different input methods were identified by the authors. Which of the following is one of those
input methods?
A. EDI
B. ERP
C. EAI
D. EPS

73. Source documents are usually preprinted and sequentially prenumbered. Which of the following is not
one of the reasons for this prenumbering and preprinting?
A. To have an established format to capture data
B. To assure that there is no duplicate source documents
C. To be sure that all of the documents have been recorded
D. To be sure that all of the documents are accounted for

74. This method of input for AIS is considered to be time consuming and error prone due to the human
effort required to write on some documents and to manually key in the data:
A. Bar Coding
B. Key Punching
C. Source Documents and Keying
D. Point of Sale Systems

75. The use of employee prepared time cards and the entering of the time worked by the payroll
department is a good example of which type of input method for AIS?
A. EDI
B. Bar Coding
C. Point of Use System
D. Source Documents and Keying

76. A printed code consisting of a series of vertical, machine readable, rectangular bars and spaces that
vary in width and are arranged in a specific ways to represent letters and numbers are referred to as:
A. COBOL Coding
B. Bar Coding
C. Key Coding
D. EOS Coding

77. Bar codes can be used in a number of different instances for input:
A. Products that are sold
B. Identification of individuals
C. Track work in process
D. Prepare financial statements

78. A method of using hardware and software that captures retail sales transactions by standard bar
coding is referred to as:
A. Point of Sale System
B. COBOL System
C. Inventory Tracking System
D. UPC System
79. The letter UPC, when relating to bar codes, stands for:
A. United Price Code
B. Universal Product Code
C. Unity Product Cost
D. Ulterior Price Company

80. The inter-company, computer-to-computer transfer of business documents in a standard business


format is referred to as:
A. Electronic Data Interchange
B. E-Business Document Identification
C. Earned Daily Interest
D. Electronic Document Idea

81. The major difference between EDI and e-business is:


A. EDI uses the internet and e-business uses dedicated networks
B. EDI does not require the use of computers
C. EDI uses dedicated networks and e-business uses the internet
D. There is no difference between EDI and e-business

82. After the accounting information has been input into the accounting system, it must be processed.
Process accounting data involves:
A. Source documents and keying
B. Bar codes and point of sale
C. Electronic data interchange and e-business
D. Calculations, classification, and summarization.

83. In a manual accounting system, the processing of accounting information includes:


A. Recording in the ledger
B. Posting to the journal
C. Closing the necessary ledger accounts
D. Adjusting the journal

84. When all similar transactions are grouped together for a specified time and then processed together
as a group, the process is referred to as:
A. Grouped processing
B. Batch processing
C. Bound processing
D. Unit processing

85. An organization that has applications having large volumes of similar transactions that can be
processed at regular intervals is best suited to use which method of processing?
A. Real-time processing
B. Point of sale processing
C. Batch processing
D. Sequential processing

86. Which of the following is not one of the advantages of batch processing?
A. It is generally easier to control than other types of computerized systems.
B. It uses less costly hardware and software.
C. It is very efficient for large volumes of like transactions when most items in the master file are used.
D. Information can be provided to users on a timely basis.

87. Which of the following is one of the disadvantages to batch processing?


A. The hardware and software are more expensive.
B. Integration across business processes is difficult in legacy systems that are batch oriented.
C. Batch systems can be difficult to audit because of the complexity of the system.
D. Errors can be corrected immediately because the system checks for input errors as the data is
entered.

88. The processing system where transactions are processed immediately and where output is available
immediately is referred to as:
A. Real-time processing
B. Online processing
C. Batch processing
D. Sequential processing

89. The advantages to real-time processing include:


A. As the data are entered, the system checks for input errors - therefore, errors can be corrected
immediately.
B. Information is provided to users on a timely basis.
C. All files are constantly up-to-date.
D. All of the above are advantages.

90. The disadvantages to real-time processing include:


A. Processing can take longer than normal if the master files are large.
B. Adding or deleting records takes a considerable amount of time.
C. The single database that is shared is more susceptible to unauthorized access.
D. Data duplication is likely because each process uses its own master file.

91. Many different types of output are generated by an accounting information system. The authors
identified general categories of output. The category that would include credit memorandums, receiving
memorandums, and production schedules is:
A. Internal documents
B. Internal reports
C. External reports
D. Trading partner documents

92. Many different types of output are generated by an accounting information system. The authors
identified general categories of output. The category that would include any document that management
determines would be useful to the business is:
A. Internal documents
B. Internal reports
C. External reports
D. Trading partner documents

93. Documentation of the accounting system allows:


A. The accountant to analyze and understand the procedures and business process and the systems that
capture and record the accounting data.
B. The non-accountant to create a picture or chart of what should happen within the accounting system.
C. The investor to see inside the accounting system so that he / she can better understand the financial
statements.
D. The accountant to determine which financial statements will be necessary to properly report the results
of operations.

94. Which of the following is not one of the popular documentation methods for processes and systems
presented in the textbook?
A. Process maps
B. Document flowcharts
C. Entity relationship diagram
D. Document creation map

95. A pictorial representation of business processes in which the actual flow and sequence of events in
the process are presented in the diagram form - the start of the process, the steps within the process, and
the finish of the process is referred to as:
A. System flowchart
B. ER Diagram
C. Process Map
D. Data Flow Diagram

96. Which of the process map symbols is used to show the start and / or finish of a process?
A. Rectangle
B. Oval
C. Diamond
D. Circle

97. Which of the process map symbols is used to show a task or activity in the process?
A. Rectangle
B. Oval
C. Diamond
D. Circle

98. Which of the process map symbols is used to show a point in the process when a decision must be
made?
A. Rectangle
B. Oval
C. Diamond
D. Circle

99. A process map shows a circle with a letter or number in the middle. This symbol is used to show:
A. That there is a break in the process.
B. That there is a decision to be made.
C. That a process is starting.
D. That there is an activity that will take place.
100. This method of system documentation is intended to show the entire system, including inputs
manual and computerized processes, and outputs.
A. Procedure mapping
B. System flowcharting
C. Data flow diagramming
D. Entity relationship diagrams

1. Which of the following database models is considered to be the most versatile?

A. The hierarchical model.


B. The tree model.
C. The relational model.
D. The network model.

2. Compared to batch processing, real-time processing has which of the following advantages?

A. Ease of implementation.
B. Ease of auditing.
C. Timeliness of information.
D. Efficiency of processing.

3. Which one of the following input controls or edit checks would catch certain types of errors within
the payment amount field of a transaction?

A. Echo check.
B. Record count.
C. Limit check.
D. Check digit.

4. DB2, Oracle, SQL Server, and Access are

A. Library systems.
B. Access control systems.
C. Programming languages.
D. Database management systems.

5. Control objectives regarding effectiveness and efficiency, reliability, and compliance are the basis
of which control framework?

A. eSAC.
B. COSO.
C. GTAG.
D. COBIT.
6. Which of the following computerized control procedures would be most effective in ensuring that
data uploaded from desktop computers to a server are complete and that no additional data are
added?

A. Field-level edit controls that test each field for alphanumerical integrity.
B. Passwords that effectively limit access to only those authorized to upload the data to the mainframe
computer.
C. Batch control totals, including control totals and hash totals.
D. Self-checking digits to ensure that only authorized part numbers are added to the database.

7. What technique could be used to prevent the input of alphabetic characters into an all-numeric
identification number?

A. A format check.
B. A record count.
C. A check digit.
D. A sequence check.

8. Passwords for personal computer software programs are designed to prevent

A. Unauthorized access to the computer.


B. Unauthorized use of the software.
C. Inaccurate processing of data.
D. Incomplete updating of data files.

9. A small client recently put its cash disbursements system on a server. About which of the
following internal control features would an auditor most likely be concerned?

A. The server is operated by employees who have cash custody responsibilities.


B. Programming of the applications are in Visual Basic rather than Java.
C. Only one employee has the password to gain access to the cash disbursement system.
D. There are restrictions on the amount of data that can be stored and on the length of time that data can
be stored.

10. When assessing application controls, which one of the following input controls or edit checks is
most likely to be used to detect a data input error in the customer account number field?

A. Hash total.
B. Control total.
C. Validity check
D. Limit check.
11. A firm has recently converted its purchasing cycle from a manual process to an online computer
system. Which of the following is a probable result associated with conversion to the new
automatic system?

A. Traditional duties are less segregated.


B. The firm's risk exposures are reduced.
C. Processing errors are increased.
D. Processing time is increased.

12. Which of the following types of controls is not described in the IT Governance Institute's Control
Objectives for Information and Related Technology (COBIT)?

A. Business controls.
B. Process controls.
C. General controls.
D. Exchange controls.

13. A database is

A. A collection of related files.


B. Essential for the storage of large data sets.
C. A real-time system.
D. A network of computer terminals.

14. Which of the following is considered an application input control?

A. Edit check.
B. Report distribution log.
C. Exception report.
D. Run control total.

15. The concept of timeliness of data availability is most relevant to

A. Online systems.
B. Manual systems.
C. Computerized systems.
D. Payroll systems.

16. The most common computer-related problem confronting organizations is

A. Input errors and omissions.


B. Fraud.
C. Disruption to computer processing caused by natural disasters.
D. Hardware malfunction.

17. Which of the following is the intended users of control frameworks such as COBIT?

A. Everyone in the organization.


B. Senior management only.
C. Internal and external auditors only.
D. Anyone with IT control responsibilities.

18. A logical view of an entire database is a

A. Subschema.
B. Network.
C. Schema.
D. Hierarchy.

19. The risks created by rapid changes in IT have not affected which concepts of internal control?
1. Cost-benefit analysis
2. Control environment
3. Reasonable assurance
4. Management's responsibility

A. 3 and 4 only.
B. 1, 2, 3, and 4.
C. 2, 3, and 4 only.
D. 1 and 2 only.

20. Internal control objectives remain essentially the same although technology, risks, and control
methods change. Thus, many concepts of control (management's responsibility, the role of the
control environment, reasonable assurance, monitoring, and cost-benefit analysis) are relevant
regardless of IT changes.

21. Which of the following characteristics distinguishes computer processing from manual
processing?

A. The potential for systematic errors is ordinarily greater in manual processing than in computerized
processing.
B. Computer processing virtually eliminates the occurrence of computational error normally associated
with manual processing.
C. Errors or fraud in computer processing will be detected soon after their occurrence.
D. Most computer systems are designed so that transaction trails useful for audit purposes do not exist.

22. Application control objectives do not normally include assurance that


A. Processing results are received by the intended user.
B. Transaction data are complete and accurate.
C. Authorized transactions are completely processed once and only once.
D. Review and approval procedures for new systems are set by policy and adhered to.

23. An auditor anticipates assessing control risk at a low level in a computerized environment. Under
these circumstances, on which of the following procedures should the auditor initially focus?
a. programmed control procedures
b. application control procedures
c. output control procedures
d. general control procedures

24. Which of the following statements about the assessment of the risks of material misstatement in a
client's computer environment is true?
a. the auditor's objectives with respect to the assessment of the risks of material misstatement are the
same as the manual system
b. the auditor must obtain an understanding of internal control and test controls in computer environments
c. if the general controls are ineffective the auditor ordinarily can assess the risks of material
misstatement at a low level if the application controls are effective
d. the auditor usually can ignore the computer system if she can obtain an understanding of the controls
outside the computer system

25. Many customers, managers, employees, and suppliers have blamed computers for making
errors. In reality, computers make very few mechanical errors. The most likely source of errors in
a fully operational computer based system is...
a. operator error
b. systems analysis and programming
c. processing
d. input

26. When an auditor tests a computerized accounting system, which of the following is true of the test
data approach?
a. several transactions of each type must be tested
b. test data must consist of all possible valid and invalid conditions
c. the program tested is different from the program used throughout the year by the client
d. test data are processed by the client's computer programs under the auditor's control

Some of the more important controls that relate to accounting information systems are validity checks,
limit checks, field checks, and sign tests. These are classified as...
a. control total validation routines
b. data access validation routines
c. output controls
d. input validation routines

When evaluating internal control of an entity that processes sales transactions on the Internet, an auditor
would be most concerned about the...
a. lack of sales invoice documents as an audit trail
b. potential for computer disruptions in recording sales
c. inability to establish an integrated test facility
d. frequency for archiving and data retention

The two broad groupings of information systems control activities are general controls and application
controls. General controls include controls
a. designed to ensure that only authorized users receive output from processing
b. for developing, modifying, and maintaining computer programs
c. relating to the correction and resubmission of faulty data
d. designed to ensure that all data submitted for processing have been properly authorized

One of the major problems in a computer system is that incompatible functions may be performed by the
same individual. One compensating control is the use of...
a. echo checks
b. a self-checking digit system
c. computer generated hash totals
d. a computer access log

A management information system


C.) Supports the operations, management, and etc

Early decision models used with structured decisions...


A.) Decision Support Systems

The first phase in the evolutionary development of information system occurs when
D.) The growth of an enterprise brings about the neeed for improved admin planning and control

Who is ultimately responsible for the implementation of cost effective controls in an automated system?
B.) Operating Management

The most common computer related problem confronting organizations is


b.) input errors and ommissions

Which one of the following statements about an AIS Systems is false?


C.) An AIS is best suited to solve problems where there is great uncertainty and ill defined reporting
requirements

Which of the following risks are greater in computerized systems than in manual systems?
C.) I, II, IV

Management Activities can be classified in 3 levels:


D) Wide Scope, aggregate, future oriented information

Compared with closed systems, open systems are characterized by


A.) Less expensive Components

Which of the following is not an element of hardware?


B.) Application programs
Which of the following is a hardware device not usually associated with input?
A.) Printer

Which of the following computer hardware devices allowed for an immediate update of merchansise
inventory in a retail environment?
D.) POS terminal

Banks are required to process many transactions from paper documents...


D.) Magnetic Ink character recognition

A piece of hardware that takes the computers digital information and transforms it into signals that can be
sent over ordinary communications?
D.) Modem

Uninterruptable power supplies are used in computer centers to reduce the liklihood of ?
B.) Losing data stored in main memory

When evaluating the downsizing of the plant materials inventory systems, data center personnel...; One
reason to use RAID is to insure...
A. ) If one drive fails, all data can still be reconstructed

The location in the CPU where data and programs are temporarily stored during processing is the
C.) RAM

In a computer system, the place wher parts of the operating system program and language translator
program are permanently stored is
A.) ROM

Access time in relation to computer processing is the amount of time is takes to?
D.) Retrieve data from memory

Which of the following measures indicates the computational power of the microproccessor?
C. ) Number of bits processed per second

A manufacturer is considering using bar code ID for recording information on parts used by the
manufacturer
B.) The movement of parts is easily and quickly recorded

A highly confidential file needs to be properly deleted from a computer. The best way to acomplish this
task is to use a?
C.) Disk Utility

Several language interaces exist in a database management systems..


A.) DDL

Computer manufacturers install software programs permanently inside the computer.... this concept is
known as
C.) Firmware
Software offered to users on a "try before you buy" basis is called
A. ) Shareware

A program that edits a group of source language statements for syntax errors and translates the states
into an object program is a
B.) Compiler

A computer program produces periodic payrolls and reports. The program is a


B.) Application program

Specialized programs that performed generalized functions, such as sorting and data comparison are
called
A.) Utility programs

BASIC, FORTAN, and COBOL are all examples of


C.) High level languages

SQL is best defined as


D.) Data manipulation

Regardless of the language in which an application program is written, its execution by a computer
requires that primary memory contain?
B.) An Operating System

Computers understand codes that represent letters of the alphabet, numbers or special characters. These
codes require that data be converted into predefined groups of binary digits . Such chains of digits are
referred to as
D.) Bytes

Computer containing more than one central processing unit are increasingly common. This feature
enables a computer to execute multiple instructions from multiple programs simulataneously. This is
known as
C.) Multiprocessing

The primary purpose of a macro program in a spreadsheet application is to allow the end user to
A.) Reduce keystrokes

Structured programming is best described as a technique that


B.) reduces the maintanence time of programs by the use of small scale program modules

In an inventory system on a database management system, one stored record contains part number, part
name, part color and part weight, The individual items are known as
A.) Fields

Block Codes
C.) Allow a user to assign meaning to particular segments of a coding scheme
An inventory clerk using a computer terminal views the following on screen: collectively these data make
up
D) Record

A file containing relatively permanent infomraino used as a source of reference and periodically updated
with detail is termed a
C. Master file

1,089,000
1,089,000

The relationship between online, real time, database systems nd batch processing systems is that a firm
C.) May use both processing modes concurrently

Sequential Access means that


D.) To read record 500, records 1 through 499 must be read first

Which of the following features is least likely to be found in a real time application?
D.) Turnaround Documents

Data Processing assigns a unique Identification code or key to each data record. Which one of the
following staements about coding is false?
C.) Secondary Keys are used when the primary keys cannot be found.

A business uses magnetic disks to store accounts receivable information. What data file concepts
underlie the ability to answer customer inquiries as they are received?
C.) Record keys, indexes, and pointers.

Bunch of States listed in a table


C.) column 3 only

Which of the following is most likely characteristic of a direct access file that uses indexes or dictionaries
as its addressing technique when processing randomly?
B.) Two access are required to retrieve each record

A system updates master files in batch mode overnight and duplicated copies of the files are updated and
queried during the day. Which of the following best describes this online system?
A.) memo updating

Of the following, the greatest advantage of a database server architecture is


A.) data reduncacy can be reduced

One advantage pf a database management system is


D.) The independence of the data from the application programs, which allows the programs to be
developed for the users specific needs without concern for data capture problems

Which of the following is a false statemt about a database management systems applcation environment?
B.) Data are shared by passing files between programs or systems
An organization uses a DBMS as a repository for data.
B.) Concurrency update controls are in place
In a database system, locking of data helps presernve data integrity by permitting transactions to have
control of all the data needed to complete the transactions however, implementing a locking procedure
could lead to
D.) Deadly embraces

The selected option is unlikely to use a heriarchichal database system because


B.) Programming queries for it are too costly and time consuming

The limiting factor in the brand managers use of relational database system to answer their ad hoc
questions will most likely be
B.) Obtaining computer resources for complicated queries

The organizations senior management was pleased that its brand new managers were taking the
inititative to use sales data creatively
A.) A source code review of the queries

The brand managers tried to import the POS data into personal computer spreadsheets for analysis.
Their efforts were unsuccessful most likely because
A.) The complexity of the mainframe data structure and the large volume of data

Evenutally the brand managers chose a data analysis tool and report writer that permitted
mulitdimensional views of the data
D.) Criteria for retrieving the data

After abandoning spreadsheets as their analysis tool,


C.) Joining the two tables on product and store identification codes

An internal auditor encounters a batch processed payrooll in which each record contains,
A.) Single Flat file structure

A flat file structure is used in database management systems when a


D.) Relational database model is selected for use

To trace data through several application programs an auditor needs to know what programs use the data
A.) Data dictionary

All of the following are methods for distributing a relational database across multiple servers except
C.) Normalization

Which of the following database modules is considered to be the most versatile>


D.) Relational model

Users makning queries in a relational database often need to combine several tables to get the info they
want. One approach to combining tables is known as
A.) Joining

An electronic meeting conducted among several parties at remote sites is referred to as


D.) Teleconferencing

Which of the following statements is true regarding TCP/IP


C.) Every site connected to a TCP/IP Network has a unique address

Which of the following is considered to be a server local are network?


B.) A device that stores program and data files for users of the LAN

Aduit team members can use the same database and prgrams when their personal computers share a
hard disk and printer on a local are network. Which communication device enables a personal computer
to connect ot a LAN
A.) A network interface card that plugs into the motherboard

In distributed data processing a ring network,


B.) Links all communication channels to form a loop, and each link passes communications through its
neighbor ot hte apporpriate location.

When two devices in a data communications system are communicating. there must be an agreement as
to how both data and control information are to be packaged and interpreted. Which of the following terms
is commonly used to describe this type of agreement?
D.) Communication protocol

Large organizations often have their own telecommunications networks for transmitting and receiving
voice, data and images. Very small organizations however are unlikely to be able to make the investment
required for their own networks and are more likely to use
A.) Commercial wireless and public switch lines

An insurance firm uses a wide area network to allow agents away from the home office to obtain current
rates and client infomration to submit approved claims using notebook computers and dial in programs
D.) End to End data encryption

Advantages of using fiber optic cables are that


D.) II,III,IV

A real estate brokerage firm is moving into a building that is considering the installation of a digital branch
exchange
B.) The system cannot easily handle large volumes of data

The internet conists of a series of networks that include


A.) Gateways to allow personal computers to connect to servers

Which of the following is true concerning HTML?


C.) The language is independent hardware and software

XML
A.) Is focused on the content of the data

Which of the following is false statement about XBRL?


C.) XBRL is used primarily in the US
The firewall system that limits access to a computer network by routing users to replicated web pages is
C.) A proxy server

An internet firewall is designed to provide adequate protection against which of the following?
B.) Unauthenticated logins from outside users

Which of the following statements is true regarding electronic mail security?


C.) I and III

The most difficult aspect of using internet resources is


B.) Locating the best information source

The proper sequence of activties in the systems development life cycle is


C.) Analysis, design, implementation and operation

An MIS manager has only enough resources to install either a new payroll system or a new data security
system but not both. Which of the following actions is most appropriate?
D.) Having the information systems steering committee set the priority

Which of the following is not considered a typical risk associated with outsourcing?
D.) Less availability of expertise

The strengths of the bottoms up approach ti systems development are that it


A.) I and III

Enterprise resource planning software packages, such as SAP and Oracle, are all inclusive systems that
attempt to provide advantages to the auditor because they
C.) Typically have built in transaction logs and ability to produce a variety of diagnostic reports

Using telecommunications provider affects in house networks. To prepare for changes resulting from
enhanced external network services, management should
A.) Optimize in house networks to avoid bottlenecks that would limit the benefits offered by the
telecommunications provider

Even though an organization is commited to using its mainframe for its manufacturing plant operations it
has been looking for wys to downsize other applications. The purpose of downsizing is to
D.) Decrease Costs

Many companies and government organizations would like to convert to open systems in order to
C.) Use less expensive computing equipment

The implementation of a client server architecture offers significant potential benefits but also introduces
certain management and control issues. For this architecture which of the following is true?
D.) the diagnosis and resolution of system probelms may be difficult because client server systems can
rarely be constructed without products from a number of vendors
Object technology provides a new and better way of enabling developers and users to build and tailor
applications. In light of this technology, what action should management take with respect to its older
system?
C.) investigate the integration of object based capabilities with legacy systems
Object technology has become more important in companies strategic use of information systems
because of its potential to
A.) Permit quicker and more reliable development of systems

An electronics firm has decided to implement a new system through the use of rapid application
development techniques. Which of the following should be included in the development of the new
system?
C.) Creating the system module by module until completed

Which of the following risks is more likely to be encountered in an end user computer environment as
compared with a mainframe computer system?
C.) Applications that are difficult to integrate with other information systems

Two mahor retail companies, both publicly traded and operating in the same geographic area, have
recently merged.
A.) The number of programmers and systems analysts employed by each company

A systems development approach used to quickly produce a model of user interfaces, user interactions
with the system, and process logic is called
B.) Prototyping

If the slow response times were caused by application software the most likely cause would be
application programs that
C.) invoke more input-output operations than are necessary for specified functions

If the response time is caused by central site hardware, the most likely cause is

In the consideration of internal control, the auditor is basically concerned that it provides reasonable
assurance that:
A. Management can not override the system.
B. Operational efficiency has been achieved in accordance with management plans.
C. Misstatements have been prevented or detected.
D. Controls have not been circumvented by collusion.

2. Proper segregation of duties reduces the opportunities to allow any employee to be in a position to both
A. Journalize cash receipts and disbursements and prepare the financial statements.
B. Monitor internal controls and evaluate whether the controls are operating as intended.
C. Adopt new accounting pronouncements and authorize the recording of transactions.
D. Record and conceal fraudulent transactions in the normal course of assigned tasks.

3. Which one of the following would the auditor consider to be an incompatible operation if the cashier
receives remittances from the mailroom?
A. The cashier prepares the daily deposit.
B. The cashier makes the daily deposit at a local bank.
C. The cashier posts the receipts to the accounts receivable subsidiary ledger.
D. The cashier endorses the checks.

4. Properly designed internal control will permit the same employee to:
A. Receive and deposit checks, and also approve write-offs of customer accounts.
B. Approve vouchers for payment, and also receive and deposit cash.
C. Reconcile the bank statements, and also receive and deposit cash.
D. Sign checks, and also cancel supporting documents.

5. Which of the following fraudulent activities most likely could be perpetrated due to the lack of effective
internal controls in the revenue cycle?
A. Merchandise received is not promptly reconciled to the outstanding purchase order file.
B. Obsolete items included in inventory balances are rarely reduced to the lower of cost or market value.
C. The write-off of receivables by personnel who receive cash permits the misappropriation of cash.
D. Fictitious transactions are recorded that cause an understatement of revenue and overstatement of
receivables.

A manufacturer is considering using bar code identification for recording information on parts used by the
manufacturer. A reason to use bar codes rather than other means of identification is to ensure that
A. Vendors use the same identification methods.
B. The movement of parts is easily and quickly recorded.
C. The movement of all parts is recorded.
D. Vendors use the same part numbers.

Enterprise resource planning (ERP) software packages, such as SAP ERP Central Component and
Oracle e-Business Suite, are all-inclusive systems that attempt to provide entity-wide information. ERP
systems provide advantages to an organization's auditors because they
A. Have proven difficult for some firms to install.
B. Have been installed by smaller firms so, to date, few auditors have encountered them.
C. Typically have built-in transaction logs and ability to produce a variety of diagnostic reports.
D. Typically require firms to reduce the division of duties and responsibilities found in traditional systems.

Control activities constitute one of the five components of internal control described in the COSO model.
Control activities do not encompass
A. Information processing.
B. Physical controls.
C. An internal auditing function.
D. Performance reviews.

In a large organization, the biggest risk in not having an adequately staffed information center help desk is
A. Persistent errors in user interaction with systems.
B. Increased likelihood of use of unauthorized program code.
C. Inadequate documentation for application systems.
D. Increased difficulty in performing application audits.

Misstatements in a batch computer system caused by incorrect programs or data may not be detected
immediately because
A. Errors in some transactions may cause rejection of other transactions in the batch.
B. The processing of transactions in a batch system is not uniform.
C. The identification of errors in input data typically is not part of the program.
D. There are time delays in processing transactions in a batch system.

In order to obtain an initial understanding of internal control sufficient to assess the risk of material
misstatement of the financial statements, an auditor would most likely perform which of the following
procedures?
A. Tests of key controls to determine whether they are effective.
B. Analytical procedures to determine the need for specific controls.
C. Risk-assessment procedures to evaluate the design of relevant controls.
D. Expanded substantive testing to identify relevant controls.

nternal controls are designed to provide reasonable assurance that


A. Management's plans have not been circumvented by worker collusion.
B. The internal auditing department's guidance and oversight of management's performance is
accomplished economically and efficiently.
C. Management's planning, organizing, and directing processes are properly evaluated.
D. Material errors or fraud will be prevented, or detected and corrected, within a timely period by
employees in the course of performing their assigned duties.

In the accounting system of Apogee Company, the quantities counted by the receiving department and
entered at a terminal are transmitted to the computer, which immediately transmits the amounts back to
the terminal for display on the terminal screen. This display enables the operator to
A. Establish the validity of the account number.
B. Verify that the amount was entered accurately.
C. Verify the authorization of the disbursement.
D. Prevent the overpayment of the account.

When erroneous data are detected by computer program controls, such data may be excluded from
processing and printed on an error report. This error report should be reviewed and followed up by the
A. Data control group.
B. Systems analyst.
C. Computer programmer.
D. Computer operator.

Data processing activities may be classified in terms of three stages or processes: input, processing, and
output. An activity that is not normally associated with the input stage is
A. Reporting.
B. Verifying.
C. Recording.
D. Batching.

A proper segregation of duties requires that an individual


A. Authorizing a transaction records it.
B. Maintaining custody of an asset be entitled to access the accounting records for the asset.
C. Authorizing a transaction maintain custody of the asset that resulted from the transaction.
D. Recording a transaction not compare the accounting record of the asset with the asset itself.

The purpose of input controls is to ensure the


A. Completeness, accuracy, and validity of input.
B. Completeness, accuracy, and validity of updating.
C. Authorization of access to data files.
D. Authorization of access to program files.
An employee in the receiving department keyed in a shipment from a remote terminal and inadvertently
omitted the purchase order number. The best systems control to detect this error is
A. Completeness test.
B. Compatibility test.
C. Sequence check.
D. Reasonableness test.

Master files maintained as part of the sales order processing system are
A. Accounts receivable and finished goods inventory.
B. Accounts payable, accounts receivable, and finished goods inventory.
C. Accounts receivable and bill of materials.
D. Accounts receivable, sales summary, and production operations list.

Internal control can provide only reasonable assurance of achieving an entity's control objectives. The
likelihood of achieving those objectives is affected by which limitation inherent to internal control?
A. The board of directors is active and independent.
B. The auditor's primary responsibility is the detection of fraud.
C. The cost of internal control should not exceed its benefits.
D. Management monitors internal control.

A mail-order retailer of low-cost novelty items is receiving an increasing number of complaints from
customers about the wrong merchandise being shipped. The order code for items has the format
wwxxyyzz. The major category is ww, xx is the minor category, yy identifies the item, and zz identifies the
catalog. In many cases, the wrong merchandise was sent because adjacent characters in the order code
had been transposed. The best control for decreasing the number of orders with the wrong merchandise
is to
A. Separate the parts of the order code with hyphens to make the characters easier to read.
B. Use a master file reference for all order codes to verify the existence of items.
C. Add check-digits to the order codes and verify them for each order.
D. Require customers to specify the name for each item they order.

If internal control is properly designed, the same employee may be permitted to


A. Receive and deposit checks and also approve write-offs of customer accounts.
B. Reconcile the bank statements and also receive and deposit cash.
C. Sign checks and also cancel supporting documents.
D. Approve vouchers for payment and also sign checks.

Which of the following best describe the interrelated components of internal control?
A. Organizational structure, management philosophy, and planning.
B. Risk assessment process, backup facilities, responsibility accounting, and natural laws.
C. Assignment of authority and responsibility, management philosophy, and organizational structure.
D. Control environment; risk assessment process; control activities; the information system, including
related business processes; and monitoring of controls.

Proper segregation of duties reduces the opportunities to allow persons to be in positions both to
A. Perpetrate and conceal fraud and error.
B. Record cash receipts and cash disbursements.
C. Establish internal control and authorize transactions.
D. Journalize entries and prepare financial statements.
In obtaining an understanding of internal control, the auditor may trace several transactions through the
control process, including how the transactions interface with any service organizations whose services
are part of the information system. The primary purpose of this task is to
A. Replace substantive procedures.
B. Determine the effectiveness of the control procedures.
C. Detect fraud.
D. Determine whether the controls have been implemented.

A document flowchart represents the


A. Possible combinations of alternative logic conditions and corresponding courses of action for each
condition in a computer program.
B. Flow of forms that relate to a particular transaction through an organization.
C. Flow of data through a series of operations in an automated data processing system.
D. Sequence of logical operations performed during the execution of a computer program.

Which of the following statements about internal control is true?


A. Exceptionally effective internal control is enough for the auditor to eliminate substantive procedures on
a significant account balance.
B. Properly maintained internal control reasonably ensures that collusion among employees cannot occur.
C. The establishment and maintenance of internal control are important responsibilities of the internal
auditor.
D. A limitation of internal control is that management makes judgments about the extent of controls it
implements.

A company updates its accounts receivable master file weekly and retains the master files and
corresponding update transactions for the most recent 2-week period. The purpose of this practice is to
A. Verify run-to-run control totals for receivables.
B. Permit reconstruction of the master file if needed.
C. Match internal labels to avoid writing on the wrong volume.
D. Validate groups of update transactions for each version.

Which of the following represents an internal control weakness in a computer-based system?


A. Computer programmers write and revise programs designed by analysts.
B. Computer operators have access to operator instructions and the authority to change programs.
C. The computer librarian maintains custody and recordkeeping for computer application programs.
D. The data control group is solely responsible for distributing reports and other output.

An entity should consider the cost of a control in relationship to the risk. Which of the following controls
best reflects this philosophy for a large dollar investment in heavy machine tools?
A. Imprinting a controlled identification number on each tool.
B. Placing security guards at every entrance 24 hours a day.
C. Having all dispositions approved by the vice president of sales.
D. Conducting a weekly physical inventory.

The primary reason to establish internal control is to


A. Provide reasonable assurance that the objectives of the organization are achieved.
B. Ensure the accuracy, reliability, and timeliness of information.
C. Safeguard the resources of the organization.
D. Encourage compliance with organizational objectives.
In assessing risks of material misstatement for purchases, an auditor vouches a sample of entries in the
voucher register to the supporting documents. Which assertion would this test of controls most likely
support?
A. Classification.
B. Accuracy.
C. Completeness.
D. Occurrence.

Which of the following are not directly involved in the revenue cycle?
A. Billing clerk.
B. Chief financial officer and controller.
C. Sales manager and the credit manager.
D. Receiving department clerk.

A university does not have a centralized receiving function for departmental purchases of books, supplies,
and equipment. Which of the following controls will most effectively prevent payment for goods not
received, if performed prior to invoice payment?
A. Invoices over a specified amount should be approved by the vice president of finance.
B. Vendor invoices should be matched with department purchase orders.
C. Names and addresses on vendor invoices should be compared to a list of department-authorized
vendors.
D. Vendor invoices should be approved by a departmental supervisor other than the employee ordering
the goods.

Cash receipts from sales on account have been misappropriated. Which of the following acts would
conceal this defalcation and be least likely to be detected by an auditor?
A. Overstating the accounts receivable subsidiary ledger.
B. Understating the sales journal.
C. Overstating the accounts receivable control account.
D. Understating the cash receipts journal.

Which of the following controls most likely would help ensure that all credit sales transactions of an entity
are recorded?
A. The billing department supervisor matches prenumbered shipping documents with entries in the sales
journal.
B. The accounting department supervisor independently reconciles the accounts receivable subsidiary
ledger to the accounts receivable control account monthly.
C. The accounting department supervisor controls the mailing of monthly statements to customers and
investigates any differences they report.
D. The billing department supervisor sends copies of approved sales orders to the credit department for
comparison to authorized credit limits and current customer account balances.

One of two office clerks in a small company prepares a sales invoice for $4,300; however, the invoice is
incorrectly entered by the bookkeeper in the general ledger and the accounts receivable subsidiary ledger
as $3,400. The customer subsequently remits $3,400, the amount on the monthly statement. Assuming
there are only three employees in the department, the most effective control to prevent this type of error is
A. Requiring the bookkeeper to perform periodic reconciliations of the accounts receivable subsidiary
ledger and the general ledger.
B. Requiring that monthly statements be prepared by the bookkeeper and verified by one of the other
office clerks prior to mailing.
C. Using predetermined totals to control posting routines.
D. Assigning the second office clerk to independently check the sales invoice prices, discounts,
extensions, and footings and to account for the invoice serial number.

A receiving department receives copies of purchase orders for use in identifying and recording inventory
receipts. The purchase orders list the name of the vendor and the quantities of the materials ordered. A
possible error that this system could allow is
A. Delay in recording purchases.
B. Overpayment for partial deliveries.
C. Payment to unauthorized vendors.
D. Payment for unauthorized purchases.

Which of the following internal control activities would an entity most likely use to assist in satisfying the
completeness assertion related to long-term investments?
A. The CFO vouches the acquisition of securities by comparing brokers' advices with canceled checks.
B. The controller compares the current market prices of recorded investments with the brokers' advices
on file.
C. The internal auditor compares the securities in the bank safe-deposit box with recorded investments.
D. Senior management verifies that securities in the bank safe-deposit box are registered in the entity's
name.

Which of the following are essential elements of the audit trail in an electronic data interchange (EDI)
system?
A. Contingency and disaster recovery plans.
B. Message directories and header segments.
C. Network and sender-recipient acknowledgments.
D. Trading partner security and mailbox codes.

Upon receipt of customers' checks in the mail room, a responsible employee should prepare a remittance
listing that is forwarded to the cashier. A copy of the listing should be sent to the
A. Internal auditor to investigate the listing for unusual transactions.
B. CFO to compare the listing with the monthly bank statement.
C. Accounts receivable bookkeeper to update the subsidiary accounts receivable records.
D. Entity's bank to compare the listing with the cashier's deposit slip.

An auditor tests an entity's policy of obtaining credit approval before shipping goods to customers in
support of management's financial statement assertion of
A. Occurrence.
B. Rights and obligations.
C. Valuation.
D. Completeness.

Which of the following statements is true concerning the security of messages in an electronic data
interchange (EDI) system?
A. Security in the transaction phase in EDI systems is not necessary because problems at that level will
usually be identified by the service provider.
B. Message authentication in EDI systems performs the same function as separation of duties in other
information systems.
C. When confidentiality of data is the primary risk, message authentication is the preferred control rather
than encryption.
D. Encryption performed by physically secure hardware devices is more secure than encryption
performed by software.

To safeguard the assets through effective internal control, accounts receivable that are written off should
be transferred to
A. A tax deductions file.
B. A credit manager, since customers may seek to reestablish credit by paying.
C. A separate ledger.
D. An attorney for evidence in collection proceedings.

Which of the following is usually a benefit of using electronic funds transfer for international cash
transactions?
A. Creation of self-monitoring access controls.
B. Reduction of the frequency of data entry errors.
C. Improvement of the audit trail for cash receipts and disbursements.
D. Off-site storage of source documents for cash transactions.

A firm's inventory consisted of 1,000 different items, 20 of which accounted for 70% of the dollar value.
The most recent regular quarterly manual count revealed an unnecessary 2 years' supply of several of
the more expensive items. The control that would best help to correct this oversupply problem is
A. Perpetual inventory of the larger dollar value items in the inventory.
B. Use of a control total over the number of unique inventory items.
C. Limit check on the total dollar value of the inventory.
D. Use of authorizing signatures on requisitions for inventory requested by production.

Which of the following internal control activities most likely would be used to maintain accurate inventory
records?
A. A just-in-time inventory ordering system keeps inventory levels to a desired minimum.
B. Perpetual inventory records are periodically compared with the current cost of individual inventory
items.
C. Periodic inventory counts are used to adjust the perpetual inventory records.
D. Requisitions, receiving reports, and purchase orders are independently matched before payment is
approved.

Sound internal control activities dictate that defective merchandise returned by customers be presented
initially to the
A. Sales clerk.
B. Receiving clerk.
C. Shipping department supervisor.
D. Accounts receivable supervisor.

Each of the following is an appropriate control over securities and investments except
A. Proper authorization of transactions.
B. Custodian separate from treasury function.
C. Storage in a safe-deposit box.
D. Custodian bonded and separate from investment records.

What is a possible consequence of an employee's being able to visit the safe-deposit box
unaccompanied?
A. The employee could steal securities, and the theft would never be discovered.
B. The employee could pledge corporate investments as security for a short-term personal bank loan.
C. There would be no record of when company personnel visited the safe-deposit box.
D. It would be impossible to obtain a fidelity bond on the employee.

For several years a client's physical inventory count has been lower than what was shown on the books
at the time of the count, and downward adjustments of the inventory account have been required.
Contributing to the inventory problem could be material weaknesses in internal control that led to the
failure to record some
A. Purchases returned to vendors.
B. Cash purchases.
C. Sales discounts allowed.
D. Sales returns received.

Proper authorization of write-offs of uncollectible accounts should be approved in which of the following
departments?
A. Accounts payable.
B. Credit.
C. Accounts receivable.
D. CFO.

Which of the following features is least likely to be found in a real-time application?


A. Preformatted screens.
B. Turnaround documents.
C. Automatic error correction.
D. User manuals.

For the purpose of effective internal control, postdated checks received from customers should be
A. Returned to customer.
B. Recorded as a cash sale.
C. Restrictively endorsed.
D. Placed in the joint custody of two officers.

The relationship between online, real-time database systems and batch processing systems is that
A. A firm will have only one processing mode because a single computer cannot do both.
B. A firm may use both processing modes concurrently.
C. A firm will not use batch processing if it has a large computer.
D. A firm will always prefer an online, real-time processing system because batch processing is slow.

The internal control objectives of the revenue cycle include all of the following except
A. Transactions relating to revenue are properly recorded.
B. Revenue cycle transactions are properly executed.
C. Custody over assets resulting from the revenue cycle is properly maintained.
D. Appropriate goods are ordered so that sales can be made.

Which of the following observations made during the preliminary survey of a local department store's
disbursement cycle reflects a control strength?
A. Individual department managers are responsible for the movement of merchandise from the receiving
dock to storage or sales areas as appropriate.
B. The receiving department is given a copy of the purchase order complete with a description of goods,
quantity ordered, and extended price for all merchandise ordered.
C. Individual department managers use prenumbered forms to order merchandise from vendors.
D. The CFO's office prepares checks for suppliers based on vouchers prepared by the accounts payable
department.

Which of the following statements is correct concerning internal control in an electronic data interchange
(EDI) system?
A. Control objectives for EDI systems generally are different from the objectives for other information
systems.
B. Internal controls in EDI systems rarely permit the risks of material misstatement to be assessed at an
acceptably low level.
C. Internal controls related to the segregation of duties generally are the most important controls in EDI
systems.
D. Preventive controls generally are more important than detective controls in EDI systems.

Which of the following would be the best protection for a company that wishes to prevent the lapping of
trade accounts receivable?
A. Request that customers' payment checks be made payable to the company and addressed to the
CFO.
B. Have customers send payments directly to the company's depository bank.
C. Segregate duties so that the bookkeeper in charge of the general ledger has no access to incoming
mail.
D. Segregate duties so that no employee has access to both checks from customers and currency from
daily cash receipts.

The objectives of internal control for a production cycle are to provide assurance that transactions are
properly executed and recorded, and that
A. Custody of work-in-process and of finished goods is properly maintained.
B. Transfers to finished goods are documented by a completed production report and a quality control
report.
C. Production orders are prenumbered and signed by a supervisor.
D. Independent internal verification of activity reports is established.

During the audit of a construction contract, it was discovered that the contractor was being paid for each
ton of dirt removed. The contract called for payment based on cubic yards removed. Which internal
control might have prevented this error?
A. Extension checks of invoice amounts.
B. Comparison of invoices with purchase orders or contracts.
C. Comparison of invoices with receiving reports.
D. Comparison of actual costs with budgeted costs.
Which control is most likely to give the greatest assurance that securities held as investments are
safeguarded?
A. Access to securities requires the signatures and presence of two designated officials.
B. Investment acquisitions are authorized by a member of the board of directors before execution.
C. Proceeds from the sale of investments are received by an employee who does not have access to
securities.
D. There is no access to securities between year-end and the date of the auditor's security count.

A purchasing agent places an order for inventory whenever a requisition is received from the warehouse.
The warehouse clerk issues requisitions based on periodic physical counts because no perpetual records
are maintained. Numerous duplicate orders have been placed for goods previously ordered but not
received. To prevent this excess ordering, the firm should
A. Not use purchase requisitions.
B. Count goods in the warehouse less often.
C. Use prenumbered purchase orders.
D. Keep an adequate record of open purchase orders and review it before ordering.

The procedure that best discourages the resubmission of vendor invoices after they have been paid is
A. A requirement for double endorsement of checks.
B. The mailing of payments directly to payees by accounting personnel.
C. The cancelation of vouchers by accounting personnel.
D. The cancelation of vouchers by CFO personnel.

An online bank teller system permitted withdrawals from inactive accounts. The best control for denying
such withdrawals is a
A. Duplicate record check.
B. Check-digit verification.
C. Master file lookup.
D. Proof calculation.

Cash receipts should be deposited on the day of receipt or the following business day. What is the most
appropriate audit procedure to determine that cash is promptly deposited?
A. Review cash register tapes prepared for each sale.
B. Review the functions of cash handling and maintaining accounting records for proper segregation of
duties.
C. Review the functions of cash receiving and disbursing for proper segregation of duties.
D. Compare the daily cash receipts totals with the bank deposits.

A wholesaling firm has a computerized billing system. Because of a clerical error while entering
information from the sales order, one of its customers was billed for only three of the four items ordered
and received. Which of the following controls could have prevented, or resulted in prompt detection and
correction, of this situation?
A. A completeness check that does not allow a sales invoice to be processed if key fields are blank.
B. Prenumbered shipping documents together with a procedure for follow-up any time there is not a
one-to-one relationship between shipping documents and sales invoices.
C. Matching line control counts produced by the computer with predetermined line control counts.
D. Periodic comparison of total accounts receivable per accounts receivable master file with total
accounts receivable per accounts receivable control account.
Which of the following activities is most likely to prevent the improper disposition of equipment?
A. The use of serial numbers to identify equipment that could be sold.
B. A segregation of duties between those authorized to dispose of equipment and those authorized to
approve removal work orders.
C. Periodic comparison of removal work orders with authorizing documentation.
D. A periodic analysis of the scrap sales and the repairs and maintenance accounts.

When performing an audit, a CPA notes that bad-debt expense is unusually high relative to similar firms in
the industry. The CPA should recommend which of the following controls?
A. Reconcile accounts receivable in the general ledger with the subsidiary ledger.
B. Send monthly statements of account to customers with outstanding balances.
C. Require credit checks on all new customers.
D. Use approved price lists for customer billing.

In a well-designed internal control system, employees in the same department most likely would approve
purchase orders, and also
A. Negotiate terms with vendors.
B. Authorize requisitions of goods.
C. Reconcile the open invoice file.
D. Inspect goods upon receipt.

Which of the following is an internal control weakness related to factory equipment?


A. Checks issued in payment of purchases of equipment are not signed by the controller.
B. All purchases of factory equipment are required to be made by the department in need of the
equipment.
C. Proceeds from sales of fully depreciated equipment are credited to other income.
D. Factory equipment replacements are usually made when estimated useful lives, as indicated in
depreciation schedules, have expired.

Which of the following most likely would be the result of ineffective internal control in the revenue cycle?
A. Omission of shipping documents could go undetected, causing an understatement of inventory.
B. Final authorization of credit memos by personnel in the sales department could permit an employee
defalcation scheme.
C. Fictitious transactions could be recorded, causing an understatement of revenues and an
overstatement of receivables.
D. Fraud in recording transactions in the subsidiary accounts could result in a delay in goods shipped.

When an office supply company is unable to fill an order completely, it marks the out-of-stock items as
back ordered on the customer's order and enters these items in a back order file that management can
view or print. Customers are becoming disgruntled with the company because it seems unable to keep
track of and ship out-of-stock items as soon as they are available. The best approach for ensuring prompt
delivery of out-of-stock items is to
A. Implement electronic data interchange with supply vendors to decrease the time to replenish inventory.
B. Match the back order file to goods received daily.
C. Increase inventory levels to minimize the number of times that stockouts occur.
D. Reconcile the sum of filled and back orders with the total of all orders placed daily.
.
Which of the following is a step in an auditor's decision to rely on internal controls?
A. Document that the additional audit effort to perform tests of controls exceeds the potential reduction in
substantive testing.
B. Perform tests of details of transactions and account balances to identify potential fraud and error.
C. Identify specific controls that are likely to prevent, or detect and correct, material misstatements and
perform tests of controls.
D. Apply analytical procedures to both financial data and nonfinancial information to detect conditions that
may indicate weak controls.

Alpha Company uses its sales invoices for posting perpetual inventory records. Inadequate internal
control over the invoicing function allows goods to be shipped that are not invoiced. The inadequate
controls could cause an
A. Overstatement of revenues, receivables, and inventory.
B. Understatement of revenues, receivables, and inventory.
C. Overstatement of revenues and receivables and an understatement of inventory.
D. Understatement of revenues and receivables and an overstatement of inventory

Electronic data interchange (EDI) offers significant benefits to organizations, but it is not without certain
major obstacles. Successful EDI implementation begins with which of the following?
A. Purchasing new hardware for the EDI system.
B. Mapping the work processes and flows that support the organization's goals.
C. Standardizing transaction formats and data.
D. Selecting reliable vendors for translation and communication software.

A client's program that recorded receiving report information entered directly by the receiving department
on vendor shipment receipt included a reasonableness or limit test. Which of the following errors would
this test likely detect?
A. The vendor shipped the wrong item.
B. The shipment received from the vendor was past due by 2 weeks.
C. The receipt was for a shipment from an unauthorized vendor.
D. The receiving department clerk entered the quantity of the product received as 0.

Which of the following could indicate source document fraud?


A. The same item code appears on different invoices.
B. The same invoice number appears on different invoices.
C. The same customer purchase order number appears on different customer invoices.
D. The same invoice date appears on different invoices.

Which of the following control activities is not usually performed with regard to vouchers payable in the
accounting department?
A. Having an authorized person approve the voucher.
B. Controlling the mailing of the check and remittance advice.
C. Matching the receiving report with the purchase order.
D. Determining the mathematical accuracy of the vendor's invo

Which of the following internal control activities most likely would deter lapping of collections from
customers?
A. Independent internal verification of dates of entry in the cash receipts journal with dates of daily cash
summaries.
B. Authorization of write-offs of uncollectible accounts by a supervisor independent of credit approval.
C. Separation of duties between receiving cash and posting the accounts receivable ledger.
D. Supervisory comparison of the daily cash summary with the sum of the cash receipts journal entries.

Equipment acquisitions that are misclassified as maintenance expense most likely would be detected by
an internal control activity that provides for
A. Segregation of duties for employees in the accounts payable department.
B. Authorization by the board of directors of significant equipment acquisitions.
C. Independent verification of invoices for disbursements recorded as equipment acquisitions.
D. Investigation of variances within a formal budgeting system.

A company's labor distribution report requires extensive corrections each month because of labor hours
charged to inactive jobs. Which of the following data processing input controls appears to be missing?
A. Validity test.
B. Completeness test.
C. Limit test.
D. Control total.

Which of the following is usually a benefit of transmitting transactions in an electronic data interchange
(EDI) environment?
A. No need to rely on third-party service providers to ensure security.
B. A reduced need to test computer controls related to sales and collections transactions.
C. A compressed business cycle with lower year-end receivables balances.
D. An increased opportunity to apply statistical sampling techniques to account balances.

Which of the following is a standard control over cash disbursements?


A. Checks should be sequentially numbered and the numerical sequence should be accounted for by the
person preparing bank reconciliations.
B. Checks and supporting documents should be marked "Paid" immediately after the check is returned
with the bank statement.
C. Checks should be sent directly to the payee by the employee who prepares documents that authorize
check preparation.
D. Checks should be signed by the controller and at least one other employee of the company.

Which of the following internal control activities most likely justifies reducing the assessment of the risks
of material misstatement for plant and equipment acquisitions?
A. Approval of periodic depreciation entries by a supervisor independent of the accounting department.
B. Periodic physical inspection of plant and equipment by the internal audit staff.
C. The review of prenumbered purchase orders to detect unrecorded trade-ins.
D. Comparison of current-year plant and equipment account balances with prior-year actual balances.

The four major tasks that any system must perform are...
A. Input, transformation, output, and storage.
B. Input, backup, output, and storage.
C. Input, transformation, output, and maintenance.
D. Input, transformation, storage, and feedback

Information systems (IS) strategy is determined by...


A. Business needs.
B. Individual department needs.
C. The technology available.
D. Competitors' strategies.

Which of the following is (are) a type(s) of business strategy(ies)?


A. Corporate-level strategy.
B. Business-level strategy.
C. Functional-level strategy.
D. All are types of strategies.

An interactive system environment is best characterized by...


A. Data files with records arranged sequentially.
B. The procession of group of data at regular intervals.
C. Sorting the transaction file before processing.
D. The processing of data immediately on input.

Information processing made possible by a network of computers dispersed throughout an organization is


called...
A. Online processing.
B. Interactive processing.
C. Time sharing.
D. Distributed data processing.

An insurance company that has adopted cooperative processing is planning to implement new standard
software in all its local offices. The new software has a fast response time, is very user friendly, and was
developed with extensive user involvement. The new software captures, consolidates, edits, validates,
and finally transfers standardized transaction data to the headquarters server. Local managers, who were
satisfied with existing locally written personal computer application, opposed the new approach because
they anticipated...
A. Increased workloads.
B. Centralization of all processing task.
C. More accountability.
D. Less computer equipment.

At a remote computer center, management installed an automated scheduling system to load data files
and execute programs at specific times during the day. The best approach for verifying that the
scheduling system performs as intended is to...
A. Analyze job activity with a queuing model to determine workload characteristics.
B. Simulate the resource usage and compare the results with actual results of operations.
C. Use library management software to track changes to successive versions of applications programs.
D. Audit job accounting data for file accesses and job initiation/termination messages.

A commonly used measure of the activity in a master file during a specific time period is...
A. Volatility.
B. The index ratio.
C. The frequency ratio.
D. The volume ratio.

Management is concerned that data uploaded from a personal computer to the company's server may be
erroneous. Which of the following controls would best address the issue?
A. Server data should be backed up on a regular basis.
B. Two persons should be present at the personal computer when it is uploading data.

C. The data uploaded to the server should be subject to the same edits and validation routines that online
data entry would require.
D. The users should be required to review a random sample of processed data.

Using standard procedures developed by information center personnel, staff members download specific
subsets of financial and operating data as they need it. The staff members analyze the data on their own
personal computers and share results with each other. Over time, the staff members learn to modify the
standard procedures to get subsets of financial and operating data that were not accessible through the
original procedures. The greatest risk associated with this situation is that...
A. The data obtained might be incomplete or lack currency.
B. The data definition might become outdated.
C. The server data might be corrupted by staff members' updates
D. Repeated downloading might fill up storage space on staff members' personal computers.

Advanced electronic point-of-sale (POS) systems allow instant capture and transmission of information for
which purposes?
I. Instant updating of accounting records.
II. Accumulation of marketing information.
III. Tracking of information about specific customers.
IV. Facilitation of warehousing

A. I and II only.
B. III and IV only.
C. I, II, and III only.
D. I, II, III, and IV.

An accounting information system (AIS) must include certain source documents in order to control
purchasing and accounts payable. For a manufacturing organization, the best set of documents should
include...
A. Purchase requisitions, purchase orders, inventory reports of goods needed, and vendor invoices.
B. Purchase orders, receiving reports, and inventory reports of goods needed.
C. Purchase orders, receiving reports, and vendor invoices.
D. Purchase requisitions, purchase orders, receiving reports, and vendor invoices.

In a traditional ERP system, the receipt of a customer order may result in...
I. Customer tracking of the order's progress.
II. Automatic replenishment of inventory by a supplier.
III. Hiring or reassigning of employees.
IV. Automatic adjustment of output schedules.

A. I, II, and IV only.


B. I and III only.
C. III and IV only.
D. I, II, III, and IV.

Which one of the following statements about an accounting information system (AIS) is false?
A. AIS supports day-to-day operations by collecting and sorting data about an organization's transactions.
B. The information produced by AIS is made aailable to all levels of management for use in planning and
controlling an organization's activities.
C. AIS is best suited to solve problems where there is great uncertainty and ill-defined reporting
requirements.
D. AIS is often referred to as a transaction processing system

A principal advantage of an ERP system is


A. Program-data dependence.
B. Data redundancy.
C. Separate data updating for different functions.
D. Centralization of data.

Enterprise resource planning (ERP) software packages, such as SAP ERP Central Component and
Oracle e-Business Suite, are all-inclusive systems that attempt to provide entity-wide information. ERP
systems provide advantages to an organization's auditors because they...
A. Have proven difficult for some firms to install.
B. Typically require firms to reduce the division of duties and responsibilities found in traditional systems.
C. Typically have built-in transaction logs and ability to produce a variety of diagnostic reports.
D. Have been installed by smaller firms so, to date, few auditors have encountered them.

Which of the following is the best example of the use of a decision support system (DSS)?
A. A manager uses a personal-computer-based simulation model to determine whether one of the
company's ships would be able to satisfy a particular delivery schedule.
B,. An auditor uses a generalized audit software package to retrieve several purchase orders for detailed
vouching.
C. A manager uses the query language features of a database management system (DBMS) to compile a
report showing customers whose average purchase exceeds $2,500.
D. An auditor uses a personal-computer-based word processing software package to modify an internal
control questionnaire for a specific audit engagement.

The processing in expert systems is characterized by...


A. Algorithms.
B. Deterministic procedures.
C. Heuristics.
D. Simulations.

For which of the following applications would the use of a fuzzy logic system be the most appropriate
artificial intelligence (AI) choice?
A. Assigning airport gates to arriving airline flights.
B. Forecasting demand for spare auto parts.
C. Performing indoor climate control.
D. Diagnosing computer hardware problems.

Business intelligence (BI) has all of the following characteristics except...


A. Focusing on strategic objectives.
B. Giving immediate information about an organization's critical success factors.
C. Displaying information in graphical format.
D. Providing advice and answers to top management from a knowledge-based system
A(n) ____ is a type of optical disc that users can read from but not write to or erase.
DVD-ROM

A ____ overcomes the major disadvantage of CD-R disks, which is being able to write on them only once
CD-RW

____ is the process of dividing a disk into tracks and sectors, so the operating system can store and
locate data and information on the disk.
Formatting

The smallest unit of data a computer can process is a(n) ____.


Bit

____ devices are servers connected to a network with the sole purpose of providing storage.
NAS

____ means you can write on one part of a disc at one time and another part at a later time.
Multisession

ExpressCard modules can be used to add all of the following EXCEPT ____ capabilities to a computer.
Printing

____ is the number of bytes (characters) a storage medium can hold.


Capacity

True or False
Tape is used as a primary method of storage but is used most often for long-term storage and backup
False. Tape is NO LONGER

Common types of flash memory include:


a. CompactFlash(CF)
b. xD Picture Card
c. Secured Digital (SD) and Memory Stick
d. All of the Above
e. None of the Above

Users subscribe to a cloud storage service, like the kind advertised in the accompanying figure, to ____.
a. To store offsite backups of data
b. To storage large audio, video, and graphics files on the internet instantaneously
c. To allow others to access their files on the internet
d. B & A
e. All the Above
f. None of the Above

You might also like